Operadores autoadjuntos e ilimitados en QM

  1. Un operador A se dice que es autoadjunto si ( x , A ψ ) = ( A x , ψ ) por ψ , x D A y D A = D A . Pero para el operador de cantidad de movimiento de partículas libres pags ^ estos productos internos no existe, sin embargo sus valores propios son reales. Asi es pags ^ un operador auto-adjunto?

  2. ¿Por qué los operadores en mecánica cuántica en general son ilimitados ?

Pregunta relacionada por OP: physics.stackexchange.com/q/99542/2451
No puedo entender por qué dices que "para el operador de momento libre de partículas pags ^ estos productos internos no existen ". En cambio, existen y el operador de momento es autoadjunto (y no tiene valores propios, ya que su espectro es real pero puramente continuo). Los operadores de QM generalmente no están definidos en todo el espacio de Hilbert, pero solo en un dominio denso y no se puede extender en un dominio más grande que permanece simétrico.Hay un teorema importante que prueba que están acotados si y solo si el dominio es todo el espacio de Hilbert.Entonces, en general, no están acotados.
i d d X mi i k X = k mi i k X . Entonces, los valores propios son reales, como k es real. Para ψ ( X ) = mi i k X , el producto interior ψ pags ^ ψ d X = mi i k X ( k ) mi i k X d X = . Entonces diverge.
Bien, i d / d X no es el operador de cantidad de movimiento; el operador momento es la única extensión autoadjunta de i d / d X definido inicialmente en S ( R ) . En segundo lugar, mi i k X no pertenece al espacio de Hilbert (y no surgen divergencias) por lo que no es un vector propio de i d / d X y k no es un valor propio. El punto es: si uno quiere realmente entender estos temas, debe renunciar a las discusiones intuitivo-formales y debe usar las herramientas matemáticas apropiadas. De lo contrario, todo queda vago e indefinido.
Para ser más explícito, creo que tu pregunta vale y merece una respuesta, pero la respuesta no se puede dar sin el necesario rigor matemático en las definiciones al menos.

Respuestas (1)

Si H es un espacio de Hilbert complejo, y A : D ( A ) H es lineal con D ( A ) H subespacio denso, hay un único operador, el adjunto A de A satisfactorio (esta es su definición)

A ψ | ϕ = ψ | A ϕ ϕ D ( A ) , ψ D ( A )
con:
D ( A ) := { ϕ H | ϕ 1 H  con ϕ 1 | ψ = ϕ | A ψ ψ D ( A ) }
El operador densamente definido anterior A se dice que es autoadjunto si A = A . Un operador densamente definido que satisface
A ψ | ϕ = ψ | A ϕ ψ , ϕ D ( A )
se dice que es simétrica . Es claro que el adjunto de A , en este caso, es una extensión de A sí mismo.

Un operador simétrico es esencialmente autoadjunto si A es autoadjunto. Es posible probar que es equivalente a decir que A admite una extensión autoadjunta única (dada por ( A ) ).

El operador i d d X con dominio dado por el espacio de Schwartz S ( R ) (pero todo lo que sigue es válido también si el dominio inicial es C 0 ( R ) ) es simétrica y esencialmente autoadjunta. Ambas cosas i d d X y el verdadero operador de cantidad de movimiento pags := ( i d d X ) no están acotados. Ambos operadores no admiten valores propios ni vectores propios.

el espectro de pags es continua y coincide con toda la recta real.

Pasando a la transformada de Fourier-Plancherel, el operador pags := ( i d d X ) resulta coincidir con el operador multiplicativo k .

Con respecto a la cuestión de la falta de límites de la mayoría de los operadores cuánticos autoadjuntos, el punto es que un célebre teorema (una de las posibles versiones del teorema de Hellinger-Toeplitz) establece que:

un operador autoadjunto (densamente definido) A : D ( A ) H está acotado si y sólo si D ( A ) = H

y casi todos los operadores de QM, por varias razones, no están definidos en todo el espacio de Hilbert (a menos que el espacio sea de dimensión finita). Estos operadores no están inicialmente definidos en todo el espacio de Hilbert porque normalmente son operadores diferenciales. Los operadores diferenciales necesitan cierto grado de regularidad para ser aplicados en una función, mientras que el elemento genérico de un L 2 el espacio es increíblemente no regular (se define hasta conjuntos de medida cero). La subsiguiente extensión a operadores autoadjuntos explota una noción más débil de derivada (derivada débil en el sentido de Sobolev) pero el dominio más grande así obtenido es, sin embargo, muy pequeño con respecto al total. L 2 espacio.

APÉNDICE. En vista del notable comentario de Andreas Blass, creo que vale la pena enfatizar otra razón física para la falta de límites de algunos operadores autoadjuntos que representan observables en QM.

En primer lugar el espectro σ ( A ) de un observable autoadjunto representado por un operador autoadjunto A tiene el significado físico del conjunto de todos los valores posibles de lo observable. Entonces, si el observable toma un conjunto ilimitado de valores , el espectro σ ( A ) debe ser un subconjunto ilimitado de R .

En segundo lugar, si A es un operador autoadjunto (más generalmente, un operador normal), el resultado importante es cierto:

| | A | | = sorber { | λ | | λ σ ( A ) }
incluyendo los casos ilimitados donde ambos lados están + simultaneamente.

Entonces, si un observable, como pags o X o el momento angular, toma un conjunto ilimitado de valores , el auto-adjunto que lo representa tiene que ser necesariamente ilimitado (y por lo tanto definido en un subespacio denso propio del espacio de Hilbert).

Increíble respuesta. Conciso, claro, correcto y completo.
Aunque soy matemático, y aunque esta respuesta es un excelente resumen de las matemáticas relevantes, creo que vale la pena agregar una razón física por la que los operadores como el impulso deberían ser ilimitados. Si, por ejemplo, el operador pags estaban acotados, entonces esto implicaría un límite superior absoluto pags para todas las medidas posibles del momento de esta partícula. Eso no se ajusta a nuestra comprensión (clásica o cuántica) de cómo funciona el impulso. Se aplican comentarios análogos a los operadores de posición, momento angular orbital, etc.
@Andreas Blass Sí, las razones que diste son muy buenas (el espectro no está limitado, por lo que el observable no puede limitarse). Sin embargo, hay otra razón, física, para la ilimitación de pags . Es un resultado estándar que no hay pares de operadores acotados y que satisfagan las conmutaciones de la relación canónica en un dominio común como X y pags hacer. Ya que X y pags se transforman entre sí a través de un mapa unitario (transformada de Fourier-Plancherel) ninguno de ellos puede ser acotado.
@V.Moretti: En realidad, no es el espectro sino el rango numérico lo que interesa aquí.
@Freeze_S No entiendo tu comentario. Lo que escribí anteriormente (último comentario) es que el operador que representa el observable es ilimitado si y solo si su espectro, es decir, el conjunto de valores posibles del observable, es ilimitado. ¿Qué tiene de malo/impreciso?
Que su explicación podría dar una impresión errónea ya que el conjunto de valores posibles es el rango numérico en lugar del espectro. Luego, para concluir que dicho observable es autoadjunto, se puede utilizar el espectro como un paso intermedio, pero eso es solo intermedio y no tiene ningún interés desde la perspectiva física.
¿Qué quieres decir con "rango numérico"?
OK, entonces te refieres al conjunto de números ψ | A ψ con ψ D ( A ) y | | ψ | | = 1 . Sin embargo, no entiendo dónde exploté esa noción en mi respuesta. Y, hablando francamente, todavía no entiendo (lo siento) tu punto. Dijiste que "tu explicación podría dar una impresión equivocada ya que el conjunto de valores posibles es el rango numérico en lugar del espectro". En cambio, escribí (ADDENDUM) "En primer lugar, el espectro σ ( A ) de un observable autoadjunto representado por un operador autoadjunto A tiene el significado físico del conjunto de todos los valores posibles del observable"
@ValterMoretti: ¿Puede explicar un poco el teorema de Fourier-Plancherel, sobre cómo obtenemos el operador equivalente? k . Parece una transformada de Fourier y luego un producto interno, pero luego un producto interno en qué espacio, L 2 ( R ) ?
La transformada de Fourier es una transformada integral por lo que funciona en L 1 . Sin embargo, hay elementos de L 2 que no son L 1 . Las funciones de onda genéricas son de este tipo. Sin embargo, estas funciones son L 2 -límites de sucesiones de L 1 -funciones. La transformada de Fourier-Plancherel es, por lo tanto, la extensión de L 1 -Transformada de Fourier a L 2 funciones usando estas secuencias y tomando el límite obvio. Entonces no es una transformada integral sino el límite de una transformada integral que, además, resulta ser L 2 -Preservación del producto escalar.
La transformada FP se reduce a la transformada F estándar sobre funciones suficientemente regulares (funciones de Schwartz, es decir, suaves y rápidamente decrecientes con todas sus derivadas), trabajando con este tipo de funciones y usando la representación integral estándar se ve inmediatamente que i d / d X se transforma en el k -operador multiplicativo por la transformada F(-P). El resultado se extiende a todo el dominio del operador momento utilizando el hecho de que el operador es cerrado y las funciones mencionadas definen un núcleo del operador.